PT 47 section1 question 19 about polio vaccines Forum

Prepare for the LSAT or discuss it with others in this forum.
Post Reply
User avatar
walterwhite

Bronze
Posts: 178
Joined: Thu Jul 18, 2013 8:31 pm

PT 47 section1 question 19 about polio vaccines

Post by walterwhite » Tue Aug 05, 2014 7:19 pm

Isn't answer A out of scope because the stimulus argues that switching to IPV would cut down on vaccination-caused polio? I don't think the argument cares about any increase in natural polio; the stimulus maintains only that we can cut down on vaccination caused polio by switching to IPV.

When I did the question I thought that while IPV may very well lead to more natural polio, this is irrelevant because the argument focuses on cutting down vaccination-caused polio.

Thus I chose B because it demonstrates that OPV is not responsible for polio in children. Rather, children are getting polio from the vaccine because they have weak immune systems, and they would get polio regardless of whether OPV or IPV was used (weakening the argument).

Can somebody please explain why A is better than B? Thanks!


User avatar
walterwhite

Bronze
Posts: 178
Joined: Thu Jul 18, 2013 8:31 pm

Re: PT 47 section1 question 19 about polio vaccines

Post by walterwhite » Tue Aug 05, 2014 8:08 pm

yea i already read through Manhattan. none of the posts address the issue i raise, which is that the conclusion ignores natural polio and only states that IPV would result in less cases of vaccine-caused polio

User avatar
Colonel_funkadunk

Gold
Posts: 3248
Joined: Tue Mar 11, 2014 11:03 pm

Re: PT 47 section1 question 19 about polio vaccines

Post by Colonel_funkadunk » Tue Aug 05, 2014 8:15 pm

The issue is that it says that natural occurring polio cases don't occur, they are just a product of the vaccine. So even if IPV leads to half as many vaccine caused cases of polio, if this leads to more natural occurring cases of the disease then this weakens the argument. The premise is that ipv causes half as many cases as OPV---> IPV should be the main vaccine. But if the switch caused other types of polio not included in the reasoning, then this weakens the reasoning.

User avatar
DetroitRed

Bronze
Posts: 230
Joined: Sun Apr 06, 2014 8:40 am

Re: PT 47 section1 question 19 about polio vaccines

Post by DetroitRed » Tue Aug 05, 2014 9:25 pm

The issue is that it says that natural occurring polio cases don't occur, they are just a product of the vaccine. So even if IPV leads to half as many vaccine caused cases of polio, if this leads to more natural occurring cases of the disease then this weakens the argument. The premise is that ipv causes half as many cases as OPV---> IPV should be the main vaccine. But if the switch caused other types of polio not included in the reasoning, then this weakens the reasoning.
Correct. And that's essentially what Manhattan said:

"It's easy to read this piece of evidence and conclude that it makes sense to switch to the apparently safer IPV vaccine. The problem with this reasoning is that while IPV may cause fewer cases of polio, it may allow a significant number of naturally occurring cases of polio. So, overall, IPV could lead to there being an overall increase in the total number of cases of polio.

(A) weakens the argument by reminding us that the IPV vaccine could lead to increased naturally occurring cases of polio and so while there would be a reduction in vaccination-caused cases, there would still be an overall increase in the total number of cases of polio."

Want to continue reading?

Register now to search topics and post comments!

Absolutely FREE!


User avatar
Christine (MLSAT)

Bronze
Posts: 357
Joined: Fri Nov 22, 2013 3:41 pm

Re: PT 47 section1 question 19 about polio vaccines

Post by Christine (MLSAT) » Wed Aug 06, 2014 12:36 am

I think I may see the problem.

Walterwhite, when you say:
Isn't answer A out of scope because the stimulus argues that switching to IPV would cut down on vaccination-caused polio? I don't think the argument cares about any increase in natural polio; the stimulus maintains only that we can cut down on vaccination caused polio by switching to IPV.


...you are not distinguishing between the premises of the argument and it's conclusion. You're right that the premises seem oriented toward the vaccination-caused polio, but the conclusion is an altogether different animal. The conclusion is "Clearly it is time to switch from OPV to IPV." The conclusion says nothing at all about the incidence of polio, vaccination-caused OR naturally occurring.

Therefore, anything that affects whether we should or should not switch from OPV to IPV, for any good reason, will be in scope for this argument. In fact, we could have weakened this argument with an answer that said that IPV caused more cancer than OPV - that would still be 100% in scope, as it would undermine the leap to the conclusion that "we should switch."

Any bad result from IPV as it compares to OPV will weaken the argument that we should switch. Since (A) gives something bad (more natural polio with IPV than OPV), it does exactly that.

As for (B), the premises clearly state that IPV would give us only half the vaccine-caused polio cases that OPV does. We can't refute that. And (B) is referring specifically to "cases of polio caused by OPV." Putting those together, the correlation with immunodeficiency simply doesn't change the fact that IPV will cut the vaccine-caused polio numbers in half. (Also note that this answer NEVER says that the immunodeficiency caused the polio, but even if it did, it would have done so in conjuction with the OPV.)

The most important issue here is that you are reading things into the conclusion that weren't said - specifically, you are reading the conclusion to be about vaccine-caused polio, when in fact it is only saying "we should switch from OPV to IPV."

Thoughts?

User avatar
walterwhite

Bronze
Posts: 178
Joined: Thu Jul 18, 2013 8:31 pm

Re: PT 47 section1 question 19 about polio vaccines

Post by walterwhite » Wed Aug 06, 2014 2:12 am

After going over this question I screwed up question 14 of section 3 on this same prep test.

I thought that choice B was attractive because allergic reactions seems to weaken the argument in the same way choice A weakens the argument in question 19. It didn't occur to me that choice B is completely out of scope and an obvious incorrect answer choice

After looking at question 14 again however I realize it was really easy and I messed it up because I was still thinking about the polio question. I'm really in a slump with weaken questions :/

Want to continue reading?

Register for access!

Did I mention it was FREE ?


Post Reply

Return to “LSAT Prep and Discussion Forum”